The Kahlo must be the fourth painting auctioned if which one of the following is true?

Maria-Marin on July 13, 2020

Please explain between A & B

Hey!! Can you please please explain why the correct answer choice is A instead of B. I already read the other messages on the board but I don't seem to clearly get the difference between these two.

Reply
Create a free account to read and take part in forum discussions.

Already have an account? log in

shunhe on July 14, 2020

Hi @Maria-Marin,

Thanks for the question! So in this question, we’re basically being asked to find a condition that would force K to be in 4. Well first, we should notice an inference based on the game rules. We know that N has to be in 2 or 3, and we know that R has to be before N. In other words, R and N are going to be 2/3 of the first three. Now recall that V has to go before K. In other words, it’s impossible for K to be in the first three slots, since R, N, and V will always go before it. Can K be in slot 6? Also no, since K has to go before S. So K can only be in slots 4 and 5 from the beginning of the game. And that means that if anything else is in slot 5, then K is forced to be in slot 4; we see that answer choice (A) puts J in slot 5, and so (A) is the correct answer.

We can also test this out to see why it works. Let’s say we put J in 5. OK, well then V has to be in the first three slots because of rule 2. Well, as discussed, if that’s the case, it’s R, N, and V in the first three. That means we have two slots left, 4 and 6, and we know that S comes after J, so J has to go in 4, and S has to go in 6.

Now let’s take a look at (B). Does N being in third mean that K has to be in fourth? No! Consider the following:

V R N J K S

We see that this set-up doesn’t violate any of the rules, and in it, we put N in third, but K didn’t have to be in fourth. And so (B) can’t be the right answer, since we want an answer choice that forces K to be fourth.

Hope this helps! Feel free to ask any other questions that you might have.